Solving First Order Diff Eq: y^2 = x^2 + cx^3?

Click For Summary
The discussion revolves around solving the first-order differential equation dy/dx = (2x + y - 1)^2. A substitution is made with u = 2x + y, leading to a derived solution y = sqrt(2) * tan(sqrt(2) * x + C) + 1 - 2x. The user questions the textbook's answer, y^2 = x^2 + cx^3, suspecting it may be incorrect. After verifying their solution satisfies the original differential equation, they conclude that the textbook's solution does not hold. The conversation highlights that different forms of solutions can exist for differential equations, often differing by a constant.
Knissp
Messages
72
Reaction score
0

Homework Statement



dy/dx = (2x + y - 1)^2


Homework Equations





The Attempt at a Solution



Let u = 2x + y
du = 2 + dy/dx
dy/dx = du/dx - 2

dy/dx = (2x + y - 1)^2
so du/dx - 2 = (u-1)^2
du/dx = (u-1)^2 + 2
du / ((u-1)^2 + 2) = dx

1/sqrt(2) * arctan ((u-1)/sqrt(2)) = x + c

1/sqrt(2) * arctan ((2x + y -1)/sqrt(2)) = x + c

y = sqrt(2) * tan(sqrt(2) * x + C) + 1 - 2x

BUT the answer in the back of the textbook is y^2 = x^2 + cx^3. Did I mess up or is it a typo? Thank you.
 
Physics news on Phys.org
You have two answers. Check them to see if they satisfy dy/dx = (2x + y - 1)^2. If you find that your answer satisfies this DE, that's pretty good evidence that the book's answer is wrong. I don't see anything obviously wrong with your work.
 
Cool thanks my solution worked.
 
If you feel really ambitious, you could check the book's solution. Sometimes with differential equations it's possible to get what look like completely different solutions, but they both work. The key is that they differ by a constant.

As an example, sin^2(x) and -cos^2(x) look to be very different, but differ only by a constant.
 
Yep the book's sol'n sure doesn't work. Thanks for the help! :)
 
Question: A clock's minute hand has length 4 and its hour hand has length 3. What is the distance between the tips at the moment when it is increasing most rapidly?(Putnam Exam Question) Answer: Making assumption that both the hands moves at constant angular velocities, the answer is ## \sqrt{7} .## But don't you think this assumption is somewhat doubtful and wrong?

Similar threads

Replies
19
Views
2K
Replies
4
Views
2K
  • · Replies 12 ·
Replies
12
Views
3K
  • · Replies 10 ·
Replies
10
Views
2K
  • · Replies 5 ·
Replies
5
Views
2K
  • · Replies 105 ·
4
Replies
105
Views
6K
  • · Replies 4 ·
Replies
4
Views
2K
  • · Replies 14 ·
Replies
14
Views
1K
  • · Replies 1 ·
Replies
1
Views
2K
Replies
6
Views
2K